Survey
* Your assessment is very important for improving the workof artificial intelligence, which forms the content of this project
* Your assessment is very important for improving the workof artificial intelligence, which forms the content of this project
ASM Study Manual for Exam P, Fifth Edition By Dr. Krzysztof M. Ostaszewski, FSA, CFA, MAAA (krzysio@krzysio.net) Errata Effective July 5, 2013, only the latest edition of this manual will have its errata updated. You can find the errata for all latest editions of my books at: http://smartURL.it/errata Posted June 26, 2013 In the solution of Problem 4 in Practice Examination 8, the fourth item in the list of six events was mistyped as { X1 < X2 < X 3 } , while it should have been { X1 < X 3 < X2 } . Posted April 5, 2013 The second to last formula in the solution of Problem 12 in Practice Examination 5 should be: 2 2−x x=2 2 3 3 1 ⎞ ⎛3 ⎛ 3 1⎞ 1 Pr ( X > 1) = ∫ ∫ x dy dx = ∫ x ( 2 − x ) dx = ⎜ x 2 − x 3 ⎟ = ( 3 − 2) − ⎜ − ⎟ = . ⎝4 ⎝ 4 4⎠ 2 4 4 4 ⎠ x=1 1 0 1 The calculation shown was correct but there was a typo in limits of integral calculation. Posted April 2, 2013 The last formula in the solution of Problem 10 in Practice Examination 4 should be 2 +∞ 2 +∞ ⎛ 2.5 ⋅ 0.6 2.5 ⎞ ⎛ 2.5 ⋅ 0.6 2.5 ⎞ 2.5 ⋅ 0.6 2.5 5 ⋅ 0.6 2.5 E (Y ) = ∫ x ⋅ ⎜ dx + 2 ⋅ dx = dx + ∫2 ⎜⎝ x 3.5 ⎟⎠ 0.6∫ x 2.5 ∫2 x 3.5 dx = x 3.5 ⎟⎠ ⎝ 0.6 x=2 x→∞ 2.5 ⋅ 0.6 2.5 2 ⋅ 0.6 2.5 2.5 ⋅ 0.6 2.5 2.5 ⋅ 0.6 0.6 2.5 =− − = − + − 0 + ≈ 0.934273. 1.5x1.5 x=0.6 x 2.5 x=2 1.5 ⋅ 21.5 1.5 21.5 The calculation shown was correct, but a factor in the numerator of the last fraction was mistyped as 2.5, when it should have been 2. Posted March 6, 2013 In the solution of Problem 6 in Practice Examination 2, the expression rectangle [ 0, 2 ] × [ 0, 3], should be rectangle [ 0,1] × [ 0, 2 ], This expression appears twice in the solution. Posted April 23, 2012 The fourth formula in the solution of Problem 4 in Practice Examination 7 should be: 2 ± 4 − 4 ⋅ 0.0005 ⋅1850 2 ± 0.5477226 ⎪⎧ 2547.72, M= ≈ ≈⎨ 2 ⋅ 0.0005 0.001 ⎪⎩ 1452.28. intead of 2 ± 4 − 4 ⋅ 0.0005 ⋅1850 2 ± 0.5477226 ⎧⎪ 2547.72, M= ≈ ≈⎨ 0.0001 0.001 ⎩⎪ 1452.28. Posted April 20, 2012 The first formula in the last line of the solution of Problem 9 in Practice 1 1 Examination 7 should be 2 ⋅ Pr ( X ≥ 800 ) ≤ instead of 2 ⋅ Pr ( X ≥ 800 ) ≥ . 4 4 Posted March 10, 2011 The second sentence of the solution of Problem 10 in Practice Examination 1 should be: As the policy has a deductible of 1 (thousand), the claim payment is ⎧ 0, when there is no damage, with probability 0.94, ⎪ Y = ⎨ max ( 0, X − 1) , when 0 < X < 15, with probability 0.04, ⎪ 14, in the case of total loss, with probability 0.02. ⎪⎩ Posted January 15, 2011 In Problem 16 in Practice Examination 6, the calculation of the second moment of X should be: 1 3 3 E ( X 2 ) = ⋅ 0 2 + ⋅ (1+ 1) = . 4 4 Second 2 moment of T instead of E(X2 ) = E(X) = 1 2 3 3 ⋅ 0 + ⋅ (1+ 1) = . 4 4 Second moment of T 2 Posted July 24, 2010 In the solution of Problem 21 in Practice Examination 6, the statement under the first expression on the right-hand side of the third to last formula should be: number of ways to pick ordered samples of size 2 from population of size n instead of number of ways to pick ordered samples of size n − 2 from population of size n Posted July 3, 2010 In the solution of Problem 1, Practice Examination 5, at the end of the first part of the fourth sentence of the solution, 5/6 is a typo, it should be 5/36, as used in the formula for Pr (Y = 6). Posted June 9, 2010 In the alternative solution of Problem 28, Practice Examination 11, the probability tree diagram should be: 99%⋅ 25% Actuarial 25% Random student 30% 1%⋅ 25% 98.5%⋅ 30% Mathematics 1.5%⋅ 30% 45% Statistics Knows Bayes Knows no Bayes Knows Bayes Knows no Bayes 98%⋅ 45% Knows Bayes 2%⋅ 45% Knows no Bayes Some numbers in the diagram were mistyped. Posted January 5, 2010 In the description of the gamma distribution in Section 2, the condition for the range of its MGF should be t < β , not 0 < t < β . Posted June 18, 2009 The last formula in the solution of Problem 19 of Practice Examination 5 should be 1 1 1 1 instead of Pr ( X ≥ 10 ) < 2 = . Pr ( X ≥ 10 ) ≤ 2 = 4 16 4 16 Posted June 17, 2009 In the statement of Problem 9 in Practice Examination 7, Pr ( X > 800 ) should be Pr ( X ≥ 800 ) , and in the solution, all inequalities should be changed accordingly. Posted April 6, 2009 In the text of Problem 7 in Practice Examination 3, the word “whwther” should be “whether”. Posted March 19, 2009 The solution of Problem 3 in Practice Examination 8 should be: Let E be the event that a new insured is accident-free during the second policy year, and F be the event that a new insured is accident-free during the first policy year, and let G be the event that this new insured was accident-free the last year, before the policy was issued. Note that for any year only the previous year affects a given year, but not the year before that. Therefore ( ) Pr ( E ) = Pr ( E ∩ F ∩ G ) ∪ ( E ∩ F ∩ G C ) ∪ ( E ∩ F C ∩ G ) ∪ ( E ∩ F ∩ G C ) = = Pr ( E ∩ F ∩ G ) + Pr ( E ∩ F ∩ G C ) + Pr ( E ∩ F C ∩ G ) + Pr ( E ∩ F C ∩ G C ) = = Pr ( G ) ⋅ Pr ( F G ) ⋅ Pr ( E F ∩ G ) + Pr ( G C ) ⋅ Pr ( F G C ) ⋅ Pr ( E F ∩ G C ) + ( ) ( ) + Pr ( G ) ⋅ Pr F C G ⋅ Pr ( E F C ∩ G ) + Pr ( G C ) ⋅ Pr F C G C ⋅ Pr ( E F C ∩ G C ) = = 0.7 ⋅ 0.8 ⋅ 0.8 + 0.3⋅ 0.6 ⋅ 0.8 + 0.7 ⋅ (1− 0.8 ) ⋅ 0.6 + 0.3⋅ (1− 0.6 ) ⋅ 0.6 = 0.748. Answer E. Posted March 5, 2009 The first sentence of Problem 16 in Practice Examination 6 should end with t < 1, instead of t > 1. Posted March 2, 2009 The properties of the cumulant moment-generating function should be: The cumulant generating function has the following properties:ψ X ( 0 ) = 0, E ( XetX ) d tX ln E ( e ) = dt E ( etX ) t=0 tX d2 d E ( Xe ) ψ t = ( ) X dt 2 dt E ( etX ) t=0 3 ( = = E ( X ), t=0 E ( X 2 etX ) E ( etX ) − E ( XetX ) E ( XetX ) ( E ( e )) tX t=0 ) 3 d ψ X ( t ) = E ( X − E ( X )) , 3 dt t=0 but for k > 3, = Var ( X ) , 2 t=0 ( ) k dk ψ X ( t ) = ψ (Xk ) ( 0 ) < E ( X − E ( X )) . k dt t=0 Also, if X and Y are independent (we will discuss this concept later), ψ aX+b ( t ) = ψ X ( at ) + bt, andψ X+Y ( t ) = ψ X ( t ) + ψ Y ( t ) . Posted January 13, 2009 In Practice Examination 8, Problem 24, the calculation of the expected value had a typo, an extra, unnecessary p in the second line, and it instead should be: +∞ +∞ E ( X ) = ∑ k ⋅ Pr ( X = k ) = 1⋅ Pr ( X = 1) + ∑ k ⋅ Pr ( X = k ) = k=1 k=2 +∞ +∞ +∞ k=1 k=1 k=1 = p + ∑ ( k + 1) ⋅ Pr ( X = k + 1) = p + ∑ k ⋅ Pr ( X = k + 1) + ∑ Pr ( X = k + 1) = ⎛ ⎞ ⎜ ⎛ +∞ ⎟ ⎞ = p + ∑ k ⋅ (1− p ) ⋅ Pr ( X = k ) + ⎜ ⎜ ∑ Pr ( X = k + 1)⎟ − Pr ( X = 0 + 1)⎟ = ⎝ k=0 ⎠ k=1 ⎜ ⎟ =Pr( X=k+1) ⎝ this sum is equal to 1 ⎠ +∞ +∞ = p + (1− p ) ⋅ ∑ k ⋅ Pr ( X = k ) + (1− Pr ( X = 1)) = k=1 = p + (1− p ) ⋅ E ( X ) + (1− p ) = 1+ (1− p ) ⋅ E ( X ) . Posted September 1, 2008 In the solution of Problem 16 in Practice Examination 2, the left-hand side of the second formula should be E X 2 , not E ( X ) . ( ) Posted August 5, 2008 In the solution of Problem 10 in Practice Examination 8, the phrase Then the total number of claims among 10 less than $1,050 follows the binomial distribution with probability of success p = 0.8413 should be Then the total number of claims among 10 less than $1,050 follows the binomial distribution with probability of success p = 0.5793 Posted July 24, 2008 In the solution of Problem 6 of Practice Examination 7, the phrase Therefore, recalling that for a discrete random variable, whose only possible values are possible integers, should be Therefore, recalling that for a discrete random variable, whose only possible values are positive integers, Posted July 23, 2008 In the solution of Problem 5 in Practice Examination 3 the expression Var ( X2 ) = 40,000 should be Var ( X2 ) = 250,000. Posted February 7, 2008 The discussion of the lack of memory property of the geometric distribution should have the formula Pr ( X = n + k X ≥ n ) = Pr ( X = k ) corrected to: Pr ( X = n + k X > n ) = Pr ( X = k ) . Posted November 13, 2007 In the solution of Problem No. 26 of Practice Examination No. 5, the sentence: Of the five numbers, 1 can never be the median. should be Of the five numbers, neither 1 nor 5 can ever be the median. Posted October 5, 2007 In the solution of Problem No. 1 in Practice Examination No. 3, the random variable Y should refer to class B. Posted September 13, 2007 The beginning of the third sentence in the solution of Problem No. 21 in Practice Examination No. 2 should be The mean error of the 48 rounded ages, instead of The mean of the 48 rounded ages. Posted August 19, 2007 In the solution of Problem No. 4, Practice Examination No. 9, the formula ( ) ( ) = 1− Pr ({Y ≤ 1} ∩ {Y Pr Y( 5 ) > 1 = 1− Pr Y( 5 ) ≤ 1 = (1) (2) } { } { } { }) } { }) ≤ 1 ∩ Y( 3) ≤ 1 ∩ Y( 4 ) ≤ 1 ∩ Y( 5 ) ≤ 1 = = 1− Pr ({ X1 ≤ 1} ∩ { X2 ≤ 1} ∩ { X 3 ≤ 1} ∩ { X 4 ≤ 1} ∩ {Y5 ≤ 1}) = = 1− ( FX (1)) = 1− (1− e−1 ) ≈ 0.89907481. 5 5 should be ( ) ( ) = 1− Pr ({Y ≤ 1} ∩ {Y Pr Y( 5 ) > 1 = 1− Pr Y( 5 ) ≤ 1 = (1) (2) } { } { ≤ 1 ∩ Y( 3) ≤ 1 ∩ Y( 4 ) ≤ 1 ∩ Y( 5 ) ≤ 1 = = 1− Pr ({ X1 ≤ 1} ∩ { X2 ≤ 1} ∩ { X 3 ≤ 1} ∩ { X 4 ≤ 1} ∩ { X5 ≤ 1}) = = 1− ( FX (1)) = 1− (1− e−1 ) ≈ 0.89907481. 5 5 Posted August 15, 2007 In the solution of Problem No. 21, Practice Examination 8, the formula f ( x, y ) f ( x, y ) 10xy 2 10xy 2 x fX ( x Y = y) = = ∞ = y = = 2 y fY ( y ) 5y 2 2 ∫ f ( x, y )dx ∫ 10xy dx 10y ∫ x dx −∞ should be f ( x, y ) fX ( x Y = y) = = fY ( y ) ∞ 0 f ( x, y ) = 0 10xy 2 y ∫ f ( x, y )dx ∫ 10xy −∞ 0 2 dx = 10xy 2 y 10y 2 ∫ x dx = x 2x = 2 2 0.5y y 0 Posted May 15, 2007 While the solution of Problem No. 16, Practice Examination No. 10, is correct, I decided to post a better explanation of it, and here is the whole problem: May 1982 Course 110 Examination, Problem No. 18 Two balls are dropped in such a way that each ball is equally likely to fall into any one of four holes. Both balls may fall into the same hole. Let X denote the number of unoccupied holes at the end of the experiment. What is the moment generating function of X? A. 7 1 9 21 B. t + t 2 − t if t = 2 or 3, otherwise 4 2 4 8 t ⎞ 1 1 1 ⎛ 3t C. ( 3e2t + e3t ) D. ( e2t + 3e3t ) E. ⎜ e 4 + 3e 4 ⎟ 4 4 4⎝ ⎠ Solution. Each ball had four possible holes, so that the total number of ways for the balls to fall is 4 ⋅ 4 = 16. There can be only two or three holes unoccupied. If three holes are unoccupied, there are four ways to put two balls in one occupied hole. Hence, 4 1 Pr ( X = 3) = = , 16 4 3 Pr ( X = 2 ) = 1− Pr ( X = 3) = . 4 Therefore, 3 1 1 M X ( t ) = e2t + e 3t = ( 3e2t + e 3t ) . 4 4 4 Answer C. Posted May 15, 2007 In the solution of Problem No. 1, Practice Examination No. 7, both events were mistakenly labeled E1 . In order to correct that, the expression Define the following events: E1 = { X1 = 0} ∩ { X2 = 1} , E1 = { X1 = 1} ∩ { X2 = 2} . should be replaced by: Define the following events: E1 = { X1 = 0} ∩ { X2 = 1} , E2 = { X1 = 1} ∩ { X2 = 2} . Posted May 9, 2007 The first sentence of Problem No. 15 in Practice Examination No. 10 should be: A fair coin is tossed until a head appears. instead of: A fair coin is tossed until a head appears until a head appears. Posted May 9, 2007 Problem No. 10 in Practice Examination No. 9 should be as below (some of the letters τ were mistyped as r): Casualty Actuarial Society November 2005 Course 3 Examination, Problem No. 19 Claim size X follows a two-parameter Pareto distribution with parameters α and θ , for which the density and the cumulative distribution function are given below: αθ α fX ( x ) = , ( x + θ )α +1 FX ( x ) = 1− θα . ( x + θ )α 1 τ A transformed distribution Y is created by Y = X . Which of the following is the probability density function of Y ? A. fY ( y ) = τθ yτ −1 ( y + θ )τ +1 B. fY ( y ) = αθ α τ yτ −1 C. fY ( y ) = D. fY ( y ) = E. fY ( y ) = (y τ +θ ) α +1 θα θ ( y + θ )θ +1 ⎛ y⎞ ατ ⎜ ⎟ ⎝θ ⎠ τ ⎛ ⎛ y⎞τ ⎞ y ⎜ 1+ ⎜ ⎟ ⎟ ⎝ ⎝θ ⎠ ⎠ (y α +1 αθ α τ +θ ) α +1 Solution. We have X = Y τ . Therefore, fY ( y ) = f X ( x ( y )) ⋅ dx = τ yτ −1 . We conclude that dy dx αθ α αθ α τ yτ −1 τ −1 = ⋅ τ y = α +1 . τ dy ( yτ + θ )α +1 y + θ ( ) Answer B. Posted May 9, 2007 The first sentence of the solution of Problem No. 2 in Practice Examination No. 9 should be: The density of the exponential distribution with mean 1 is e− x for x > 0, while the 1 density of the exponential distribution with mean is 2e−2 x for x > 0. 2 instead of The density of the exponential distribution with mean 1 is e− x for x > 0, while the 1 density of the exponential distribution with mean is 2e− x for x > 0. 2 Posted May 9, 2007 In the solution of Problem No. 15, Practice Examination No. 3, the derivative is missing a minus, and it should 2 dX 1 − = − (8 − Y ) 3 . dY 3 The rest of the solution is unaffected. Posted May 9, 2007 Answer C in Problem No. 7 in Practice Examination No. 6 should be 0.2636, not 0.2626. It is the correct answer. Posted May 9, 2007 In Problem No. 10, Practice Examination No. 7, in the first part of the Practice Examination (not in the Solutions part), the expression E ( X ) = µY should be E ( X ) = µX . Posted May 8, 2007 Problem No. 24 in Practice Examination No. 5 should be (it had a typo in the list of possible values of x) May 1992 Course 110 Examination, Problem No. 35 Ten percent of all new businesses fail within the first year. The records of new businesses are examined until a business that failed within the first year is found. Let X be the total number of businesses examined prior to finding a business that failed within the first year. What is the probability function for X? A. 0.1⋅ 0.9 x , for x = 0,1,2, 3,... B. 0.9 ⋅ 0.1x , for x = 0,1,2, 3,... C. 0.1x ⋅ 0.9 x , for x = 1,2, 3,... D. 0.9x ⋅ 0.1x , for x = 1,2, 3,... E. 0.1⋅ ( x − 1) ⋅ 0.9 x , for x = 2, 3, 4,... Solution. Let a failure of a business be a success in a Bernoulli Trial, and a success of a business be a failure in the same Bernoulli Trial. Then X has the geometric distribution with p = 0.1, and therefore f X ( x ) = 0.1⋅ 0.9 x for x = 0, 1, 2, 3, … . Answer A. Posted May 5, 2007 Problem No. 10 in Practice Examination No. 3 should be (it had a typo in one of the integrals and I decided to reproduce the whole problem to provide a better explanation): Sample Course 1 Examination, Problem No. 35 Suppose the remaining lifetimes of a husband and a wife are independent and uniformly distributed on the interval (0, 40). An insurance company offers two products to married couples: • One which pays when the husband dies; and • One which pays when both the husband and wife have died. Calculate the covariance of the two payment times. A. 0.0 B. 44.4 C. 66.7 D. 200.0 E. 466.7 Solution. Let H be the random time to death of the husband, W be the time to death of the wife, and X be the time to the second death of the two. Clearly, X = max ( H ,W ) . 1 We have fH ( h ) = fW ( w ) = for 0 ≤ h ≤ 40, and 0 ≤ w ≤ 40. Thus 40 E ( H ) = E (W ) = 20. Furthermore, FX ( x ) = Pr ( X ≤ x ) = Pr ( max ( H ,W ) ≤ x ) = = Pr ({ H ≤ x} ∩ {W ≤ x}) = Pr ( H ≤ x ) ⋅ Pr (W ≤ x ) = x x x2 ⋅ = . 40 40 1600 This implies that x2 sX ( x ) = 1 − 1600 for 0 ≤ x ≤ 40, and w 40 ⎛ ( h,x 2w )⎞= w 40 3 120 40 80 40 max E ( X ) = ∫here 1 − dx = 40 − = − = . ⎜ 1600 ⎟⎠ 4800 3 3 3 0 ⎝ In order to find covariance, we also need to find E ( XH ) = E ( H max ( H ,W )) . max ( h, w ) = h We separate the double integral into two parts: one based on the region where the wife here lives longer and one based on the region where the husband lives longer, as illustrated in the graph below 40 h E ( H max ( H ,W )) = +∞ +∞ ∫ ∫ h max ( h, w ) ⋅ f ( h ) ⋅ f ( w ) ⋅ dwdh = H W −∞ −∞ h = 40 w = 40 ⎛ w=h 2 1 1 ⎞ ⎛ ⎞ 1 1 = ∫ ⎜ ∫ h ⋅ ⋅ dw ⎟ dh + ∫ ⎜ ∫ hw ⋅ ⋅ dw ⎟ dh = 40 40 ⎠ 40 40 ⎠ h=0 ⎝ w=0 h=0 ⎝ w=h h = 40 ⎛ h2w = ∫⎜ 1600 0 ⎝ 40 40 ⎛ ⎞ hw 2 dh + ⎟ ∫0 ⎜⎝ 3200 w=0 ⎠ w=h w = 40 w=h 40 40 ⎞ h3 1600h − h 3 dh = dh + dh = ⎟ ∫ ∫ 1600 3200 ⎠ 0 0 40 1 1 ⎛1 ⎞ ⎛1 ⎞ = ∫⎜ h+ h 3 ⎟ dh = ⎜ h 2 + h4 ⎟ ⎝2 ⎝4 3200 ⎠ 12800 ⎠ 0 h = 40 = h=0 1 1 ⋅ 40 2 + ⋅ 40 4 = 600. 4 12800 Finally, Cov ( X, H ) = E ( XH ) − E ( X ) E ( H ) = 600 − 20 ⋅ 80 2 = 66 . 3 3 Answer C. Posted April 30, 2007 Problem No. 3 in Practice Examination No. 10 should be: An insurance policy has a deductible of 10. Losses follow a probability distribution with density f X ( x ) = xe− x for x > 0 and f X ( x ) = 0 otherwise. Find the expected payment. A. e−10 B. 2e−10 C. 10e−10 D. 12e−10 E. 100e−10 Solution. Let X be the random variable describing losses, and let Y be the amount of payment. Then ⎧⎪ 0, X ≤ 10, Y =⎨ ⎩⎪ X − 10, X > 10. We can calculate the expected value of Y directly E (Y ) = +∞ ∫ ( x − 10 ) xe −x +∞ dx = 10 ∫ 10 +∞ x e dx − ∫ 10xe− x dx = 2 −x 10 u = x2 v = −e− x = du = 2xdx dv = e− x dx Integration by parts for the first integral = ( −x 2 e− x ) = x→∞ x=10 +∞ + +∞ −x −x −10 ∫ 2xe dx − ∫ 10xe dx = 100e − 10 10 −x +∞ ∫ 8xe dx = 10 x→∞ u = 8x v = −e −10 −x = 100e + 8xe −8 ( ) x=10 du = 8dx dv = e− x dx Integration by parts for the first integral −x +∞ ∫e −x dx 10 = Survival function of exponential distribution with hazard rate evaluated at x=10 = 100e−10 − 80e−10 − 8e−10 = 12e−10 . Answer D. We could also do this problem by applying the Darth Vader Rule. We have ⎧ Pr ( X > 10 ) , if y = 0, ⎪ sY ( y ) = Pr (Y > y ) = ⎨ ⎪⎩ Pr ( X − 10 > y ) , if y > 0, = u = −x v = e− x = −xe− x −x dx = −dx dv = −e x→∞ + x=10+y +∞ ⎫ ⎪ −x ⎬ = Pr ( X > y + 10 ) = ∫ xe dx, = 10+y ⎪⎭ +∞ ∫ e− x dx = 10+y Survival function of exponential with hazard rate 1 evaluated at 10+y Integration by parts = (10 + y ) e−10−y + e−10−y = (11+ y ) e−10−y for any nonnegative y. As the payment random variable is non-negative almost surely, the expected payment is +∞ ∫ (11+ y ) e −10−y dy = 11e −10 +∞ ∫e ⋅ 0 −y dy 0 This equals one because e− y for y>0 is a density of an exponential random variable with hazard rate 1 +e −10 +∞ ⋅ ∫ ye −y dy = 12e−10 . 0 This equals one because the integrand is the density in this problem. Answer D. Posted April 29, 2007 The second formula to the last in the solution of Problem No. 6 in Practice Examination No. 3 should be: 8 t 7 ⎛ ⎛ 2 + et ⎞ 8 ⎞ d 2 M (t ) d t ⎛ 2 + et ⎞ 2 t t⎛2+e ⎞ t E X = = 3e = 3e + 8e ⋅ e ⎜ ⎜ ⎟ ⎜⎝ 3 ⎟⎠ ⎟ ⎜⎝ 3 ⎟⎠ dt 2 t = 0 dt ⎝ ⎝ 3 ⎠ ⎠ t =0 ( ) = 11. t =0 instead of ( ) E X 2 d 2 M (t ) d t ⎛ 2 + et ⎞ = = 3e ⎜ dt 2 t = 0 dt ⎝ 3 ⎟⎠ 8 t =0 t ⎛ ⎛ 2 + et ⎞ 8 ⎞ t⎛2+e ⎞ t = ⎜ 3et ⎜ + 8e ⋅ e ⎟ ⎜⎝ 3 ⎟⎠ ⎟ ⎝ ⎝ 3 ⎠ ⎠ = 11. t =0 Posted April 27, 2007 In the solution of Problem No. 9 in Practice Examination No. 3, the expression max ( X,1) should be min ( X,1) . Posted April 20, 2007 Problem No. 16 in Practice Examination No. 1 should be: May 2001 Course 1 Examination, Problem No. 26, also Study Note P-09-05, Problem No. 109 A company offers earthquake insurance. Annual premiums are modeled by an exponential random variable with mean 2. Annual claims are modeled by an exponential random variable with mean 1. Premiums and claims are independent. Let X denote the ratio of claims to premiums. What is the density function of X? A. 1 2x + 1 B. 2 ( 2x + 1) C. e− x 2 D. 2e−2 x E. xe− x Solution. Let U be the annual claims and let V be the annual premiums (also random), and let fU ,V ( u, v ) be the joint density of them. Furthermore, let f X be the density of X and let FX be its cumulative distribution function. We are given that U and V are independent, and hence v − 1 −u − 2v −u 1 2 fU ,V ( u, v ) = e ⋅ e = e e 2 2 for 0 < u < ∞, 0 < v < ∞. Also, noting the graph below, v u = vx or v = u/x u we have: ⎛U ⎞ FX ( x ) = Pr ( X ≤ x ) = Pr ⎜ ≤ x ⎟ = Pr (U ≤ Vx ) = ⎝V ⎠ ∞ vx ⎛ 1 = ∫⎜− e 2 0⎝ ∞ ⎛ 1⎞ − v⎜ x + ⎟ ⎝ 2⎠ +∞ u = vx 0 u=0 ⎛ 1 −u − 2v ⎞ ∫ ⎜⎝ − 2 e e ⎟⎠ 1 −u − 2v = ∫ ∫ e e dudv = 2 0 0 ∞ vx ∫ ∫ f (u, v )dudv = U ,V 0 0 ∞ ⎛ 1 − vx − 2v 1 − 2v ⎞ dv = ∫ ⎜ − e e + e ⎟ dv = 2 2 ⎠ 0⎝ ⎛ 1⎞ v ⎛ 1 − v⎜ x + ⎟ − ⎞ 1 − 2v ⎞ + e ⎟ dv = ⎜ e ⎝ 2⎠ − e 2 ⎟ 2 ⎠ ⎝ 2x + 1 ⎠ +∞ =− 0 1 + 1. 2x + 1 Finally, f X ( x ) = FX′ ( x ) = 2 ( 2x + 1)2 . Answer B. This problem can also be done with the use of bivariate transformation. We will now give an alternative solution using that approach. Consider the following transformation X = U , Y = V. Then the inverse transformation is U = XY , V = Y . We V know that 1 −u − 2v e e 2 for u > 0, v > 0. It follows that fU ,V ( u, v ) = ∂ ( u, v ) 1 − xy − 2y ⎡ y x ⎤ 1 − xy − 2y f X ,Y ( x, y ) = fU ,V ( u ( x, y ) , v ( x, y )) ⋅ = e e ⋅ det ⎢ ⎥ = ye e ∂ ( x, y ) 2 ⎣0 1 ⎦ 2 for xy > 0 and y > 0, or just x > 0 and y > 0. Therefore, fX ( x ) = +∞ ∫ 0 1 − xy − 2y ye e dy = 2 +∞ ∫ 0 ⎛ w= 1⎞ 1 − ⎜⎝ x + 2 ⎟⎠ y ye dy = 2 1 y 2 z=− 1 ⎛ ⎜⎝ x + e 1⎞ ⎟ 2⎠ ⎛ 1⎞ −⎜ x+ ⎟ y ⎝ 2⎠ = ⎛ 1⎞ −⎜ x+ ⎟ y ⎝ 2⎠ 1 dw = dy dz = e dy 2 INTEGRATION BY PARTS ⎛ ⎞ ⎛ 1⎞ −⎜ x+ ⎟ y ⎟ ⎜ 1 1 = ⎜− y⋅ e ⎝ 2⎠ ⎟ ⎜ 2 ⎛⎜ x + 1 ⎞⎟ ⎟ ⎝ ⎝ ⎠ 2⎠ y→+∞ +∞ − ∫ 0 y= 0 ⎛ ⎞ ⎛ 1⎞ 1⎜ 1 ⎟ − ⎜⎝ x + 2 ⎟⎠ y − e dy = ⎜ ⎟ 1⎞ ⎟ 2⎜ ⎛ x+ ⎟ ⎝ ⎜⎝ 2⎠ ⎠ ⎛ ⎞ ⎛ ⎞ +∞ ⎛ 1⎞ ⎛ 1⎞ ⎟ ⎜ −⎜ x+ ⎟ y 1 1 ⎜ 1 ⎟ − ⎜⎝ x + 2 ⎟⎠ y =0+ ∫ e ⎝ 2 ⎠ dy = ⎜ ⎟ ⎜− ⎟ ⋅e 2x + 1 ⎜ 2x + 1 ⎜ ⎛ x + 1 ⎞ ⎟ ⎟ 0 ⎟ ⎜⎝ ⎟⎠ ⎝ ⎜⎝ 2⎠ ⎠ y→+∞ = 2 ( 2x + 1)2 . y= 0 Answer B, again. The second approach is probably more complicated, but it is a good exercise in the use of multivariate transformations. Posted February 19, 2007 In Problem No. 20, Practice Examination Number No. 10, answer choice is D, instead of E as stated now (typo). Posted January 20, 2007 Exercise 2.8 should read as follows: November 2001 Course 1 Examination, Problem No. 11 A company takes out an insurance policy to cover accidents that occur at its manufacturing plant. The probability that one or more accidents will occur during any 3 given month is . The number of accidents that occur in any given month is independent 5 of the number of accidents that occur in all other months. Calculate the probability that there will be at least four months in which no accidents occur before the fourth month in which at least one accident occurs. A. 0.01 B. 0.12 C. 0.23 D. 0.29 E. 0.41 Solution. Consider a Bernoulli Trial with success defined as a month with an accident, and a month 3 with no accident being a failure. Then the probability of success is . Now consider a 5 negative binomial random variable, which counts the number of failures (months with no accident) until 4 successes (months with accidents), call it X. The problem asks us to find Pr ( X ≥ 4 ) . But Pr ( X ≥ 4 ) = 1 − Pr ( X = 0 ) − Pr ( X = 1) − Pr ( X = 2 ) − Pr ( X = 3) = 4 4 4 2 4 3 ⎛ 3⎞ ⎛ 3 ⎞ ⎛ 4 ⎞ ⎛ 3 ⎞ 2 ⎛ 5⎞ ⎛ 3 ⎞ ⎛ 2 ⎞ ⎛ 6⎞ ⎛ 3 ⎞ ⎛ 2 ⎞ = 1− ⎜ ⎟ ⋅⎜ ⎟ − ⎜ ⎟ ⋅⎜ ⎟ ⋅ − ⎜ ⎟ ⋅⎜ ⎟ ⋅⎜ ⎟ − ⎜ ⎟ ⎜ ⎟ ⎜ ⎟ ≈ ⎝ 0⎠ ⎝ 5 ⎠ ⎝ 1 ⎠ ⎝ 5 ⎠ 5 ⎝ 2 ⎠ ⎝ 5 ⎠ ⎝ 5 ⎠ ⎝ 3⎠ ⎝ 5 ⎠ ⎝ 5 ⎠ ≈ 0.289792. Answer D. Posted December 31, 2006 In Section 2, the general definition of a percentile should be the 100-p-th percentile of the distribution of X is the number x p which satisfies both of ( ) ( ) the following inequalities: Pr X ≤ x p ≥ p and Pr X ≥ x p ≥ 1− p. It was mistyped as the 100-p-th percentile of the distribution of X is the number x p which satisfies both of ( ) ( ) the following inequalities: Pr X ≤ x p ≥ p and Pr X ≥ x p ≤ 1− p. Posted December 28, 2006 On page 3 in the bottom paragraph, the words: is also defined as the sent that consists of all elementary events that belong to any one of them. should be is also defined as the set that consists of all elementary events that belong to any one of them. The word “set” was mistyped as “sent.” Posted December 1, 2006 In Practice Examination 6, Problem 22, had inconsistencies in its assumptions, and it should be replaced by the following: A random variable X has the log-normal distribution with the density: 1 2 − ( ln x− µ ) 1 ⋅e 2 x 2π for x > 0, and 0 otherwise, where µ is a constant. You are given that Pr ( X ≤ 2 ) = 0.4. Find E ( X ) . fX ( x ) = A. 4.25 B. 4.66 C. 4.75 D. 5.00 E. Cannot be determined Solution. X is log-normal with parameters µ and σ 2 if W = ln X N µ, σ 2 . The PDF of the log- ( ) normal distribution with parameters µ and σ is 2 fX ( x ) = 1 − 1 ( ln x− µ ) 2 σ2 2 e xσ 2π for x > 0, and 0 otherwise. From the form of the density function in this problem we see that σ = 1. Therefore ⎛ ln X − µ ln 2 − µ ⎞ Pr ( ln X ≤ ln 2 ) = Pr ⎜ ≤ ⎟ = 0.4. ⎝ 1 1 ⎠ Let z0.6 be the 60-th percentile of the standard normal distribution. Let Z be a standard ln X − µ normal random variable. Then Pr ( Z ≤ −z0.6 ) = 0.40. But Z = is standard normal, 1 thus ln 2 − µ = −z0.6 , and µ = ln 2 + z0.6 . From the table, Φ ( 0.25 ) = 0.5987 and Φ ( 0.26 ) = 0.6026. By linear interpolation, 0.6 − 0.5987 1 z0.6 ≈ 0.25 + ⋅ ( 0.26 − 0.25 ) = 0.25 + ⋅ 0.01 ≈ 0.2533. 0.6026 − 0.5987 3 This gives µ = ln 2 + z0.6 ≈ 0.9464805. The mean of the log-normal distribution is E ( X ) = e E(X) = e Answer A. 1 0.9464805+ 2 1 µ+ σ 2 2 , so that in this case ≈ 4.2481369. Posted December 1, 2006 In Practice Examination No. 8, Problem No. 15, answer B should be listed as B. 0.1465 It was listed as B. 1465, an obvious typo. Posted November 29, 2006 The solution of Problem No. 5 in Practice Examination No. 10 is not clear in the explanation of the linear interpolation. Below is a more complete explanation: May 1982 Course 110 Examination, Problem No. 1 If a normal distribution with mean µ and variance σ 2 > 0 has 46-th percentile equal to 20σ , then µ is equal to A. 18.25σ B. 19.90σ C. 20.10σ D. 21.73σ E. Cannot be determined Solution. X−µ has the standard σ normal distribution. Let us write x0.46 for the 46-th percentile of X. We have ⎛ X − µ x0.46 − µ ⎞ ⎛ X − µ 20σ − µ ⎞ 0.46 = Pr ( X < x0.46 ) = Pr ⎜ < = Pr ⎜ < ⎟ ⎟⎠ . ⎝ σ ⎝ σ σ ⎠ σ Let X be the random variable described in the problem. Then From the table, Φ ( 0.1) = 0.5398 and Φ ( 0.11) = 0.5438. Therefore, Φ ( −0.1) = 0.4602 and Φ ( −0.11) = 0.4562. The probability desired is 0.46, and that is 0.4602 − 0.46 0.0002 1 = = 0.4602 − 0.4562 0.0040 20 of the distance between 0.4562 and 0.4602, down from 0.4602. This means that the 46-th 1 percentile is approximately of the distance between −0.1 and −0.11, down from 20 −0.1, i.e., we can get the desired percentile of the standard normal distribution via the following linear interpolation approximation: 1 0.01 z0.46 ≈ −0.1− ⋅ (( −0.1) − ( −0.11)) = −0.1− ≈ −0.1005. 20 20 20σ − µ Therefore, ≈ −0.1005, so that µ ≈ 20.1005σ . σ Answer C. Posted November 25, 2006 The third equation from the bottom in the solution of Problem No. 10 in Practice Examination No. 2 should be Pr ( A ∩ E99 ) = Pr ( A E99 ) ⋅ Pr ( E99 ) = 0.03⋅ 0.20 = 0.006. Previously it had a typo: Pr ( A ∩ E99 ) = Pr ( A E99 ) ⋅ Pr ( E99 ) = 0.03⋅ 0.20 = 0.0036. Posted November 24, 2006 In the description of the exponential distribution, the sentence The exponential distribution is related to the geometric distribution in that if we have a random variable T with exponential distribution, and we define a new random variable K as the integer portion of T (for example, if T = 7.5, then K = 7, K is simply the value of the greatest integer function when applied to T), then so defined K has the geometric distribution with p = e− λ . should be replaced by The exponential distribution is related to the geometric distribution in that if we have a random variable T with exponential distribution, and we define a new random variable K as the integer portion of T (for example, if T = 7.5, then K = 7, K is simply the value of the greatest integer function when applied to T), then so defined K has the geometric distribution with q = e− λ . The only difference is that the symbol in the last equation should be q, not p. Posted November 22, 2006 The third sentence of Problem No. 12 in Practice Examination No. 7 should read: Let X and Y be the times at which the first and second circuits fail, respectively, counting from the moment when a circuit is activated. Posted August 16, 2006 Problem No. 2 in Practice Examination No. 10 should be as below (it did not define the variable Z previously) You are given that the joint density of random variables X and Y is f X ,Y ( x, y ) = x + y for 0 < x < 1 and 0 < y < 1. What is the coefficient of variation of Z = X + Y ? A. 0.1889 B. 1.3611 C. 1.55 D. 0.50 E. 0.3194 Solution. Note that 0 < Z < 2 with probability 1. Using the concept of convolution fZ ( z ) = f X+Y ( z ) = +∞ ∫ f ( x, z − x ) dx = ∫ X ,Y −∞ ⎧ z ⎪ ∫ z dx, 0 < z ≤ 1, ⎪ 0 =⎨ 1 ⎪ z dx, 1 < z ≤ 2. ⎪ ∫ ⎩ z−1 Therefore, 1 z dx = 2 ∫ z dx = [ 0,1]∩[ z−1,z ] 0≤x≤1 0≤z−x≤1 ⎫ ⎪ ⎪ ⎧⎪ z 2 , 0 < z ≤ 1, ⎬= ⎨ ⎪ ⎪⎩ z ( 2 − z ) , 1 < z ≤ 2. ⎪ ⎭ 2 ⎫⎪ ⎧⎪ z 2 , 0 < z ≤ 1, ⎬= ⎨ 2 ⎪⎭ ⎩⎪ 2z − z , 1 < z ≤ 2. 1 1 ⎛2 1 ⎞ E ( Z ) = ∫ z dz + ∫ z ( 2 − z ) dz = + ∫ ( 2z 2 − z 3 ) dz = + ⎜ z 3 − z 4 ⎟ ⎝ 4 1 4 3 4 ⎠ 0 1 3 = z=2 = 2 z=1 1 ⎛ 16 2 1 ⎞ 1 14 3 28 31 24 7 +⎜ −4− + ⎟ = + −4 = + −4= − = , 4 ⎝ 3 3 4⎠ 2 3 6 6 6 6 6 1 2 0 1 E ( Z 2 ) = ∫ z 4 dz + ∫ z 3 ( 2 − z ) dz = 1 ⎛2 1 ⎞ = + ⎜ z4 − z5 ⎟ 5 ⎝4 5 ⎠ z=2 = z=1 2 1 + ( 2z 3 − z 4 ) dz = 5 ∫1 1 ⎛ 32 1 1 ⎞ 3 +⎜8− − + ⎟= . 5 ⎝ 5 2 5⎠ 2 Hence, 3 49 54 49 5 − = − = . 2 36 36 36 36 Finally, the coefficient of variation is 5 Var ( Z ) 5 = 36 = ≈ 0.31943828. 7 E ( Z) 7 6 Answer E. Var ( Z ) = Posted August 3, 2006 In the solution of Problem No. 4 in Practice Examination No. 1, the formula fY ( y ) = k2 ⋅ y, should be fY ( y ) = k2 ⋅1. Posted July 17, 2006 The last two lines of the solution of Problem No. 5 in Practice Examination No. 8 should be: Alternatively, d 1 d +∞ etn−1 1 +∞ etn−1 −1 −2 M X′ ( t ) = − (t − 2 ) + ⋅ ∑ = (t − 2 ) + ⋅ ∑ . dt 2 dt n=0 n! 2 n=1 ( n − 1)! Therefore, 1 1 +∞ e−1 1 1 −1 +∞ 1 1 1 3 E ( X ) = M X′ ( 0 ) = + ⋅ = + ⋅e ⋅∑ = + = . ∑ 4 2 4 ( −2 )2 2 n=1 ( n − 1)! 4 2 n=0 n! ( ) =e Posted July 3, 2006 The relationship between the survival function and the hazard rate, stated just after the definition of the hazard rate, should be x sX ( x ) = e instead of − ∫ λ X (u ) du −∞ x sX ( x ) = e ∫ − λ X ( u ) du 0 . The second equation applies in the case when the random variable X is nonnegative almost surely. Posted July 3, 2006 The condition concerning a continuous probability density function in its first definition, in Section 1, should be f X ( x ) ≥ 0 for every x ∈ instead of 0 ≤ f X ( x ) ≤ 1 for every x ∈. Posted May 16, 2006 In the solution of Problem No. 21 in Practice Examination No. 8 the last formula should be x=y y y ⎞ 1 2x 1 1 ⎛2 E X 2 Y = y = ∫ x 2 ⋅ 2 dx = 2 ⋅ ∫ 2x 3dx = 2 ⋅ ⎜ ⋅ x 4 ⎟ = y 2 . y y 0 y ⎝4 2 x=0 ⎠ 0 ( ) Posted May 16, 2006 The solution of Problem No. 13 in Practice Examination No. 3 should be (some of the exponents contained typos) Solution. Let X be the random number of passengers that show for a flight. We want to find Pr(X = 31) + Pr(X = 32). We can treat each passenger arrival as a Bernoulli Trial, and then it is clear that X has binomial distribution with n = 32, p = 0.90. Therefore, ⎛ 32 ⎞ Pr ( X = x ) = ⎜ ⋅ 0.90 x ⋅ 0.10 32−x. ⎟ x ⎝ ⎠ The probability desired is: ⎛ 32 ⎞ ⎛ 32 ⎞ Pr ( X = 31) + Pr ( X = 32 ) = ⎜ ⋅ 0.90 31 ⋅ 0.101 + ⎜ ⋅ 0.90 32 ⋅ 0.10 0 ≈ ⎟ ⎟ 31 32 ⎝ ⎠ ⎝ ⎠ ≈ 0.12208654 + 0.03433684 = 0.15642337. Answer E. Posted May 15, 2006 The solution, but not the answer, of Problem No. 30 in Practice Examination No. 9, should be changed to: Solution. First note that sX ( x ) = u = − (1− K + 0.01Ks ) v = e−0.01s +∞ −0.01s ∫ 0.01⋅ (1− K + 0.01Ks ) ⋅ e ds = x du = −0.01Kds dv = −0.01e−0.01s ds INTEGRATION BY PARTS = ( − (1− K + 0.01Ks ) ⋅ e−0.01s ) s→+∞ s=x +∞ + K ⋅ ∫ 0.01e−0.01s ds = x Survival function for exponential with hazard rate of 0.01 = (1− K + 0.01Kx ) ⋅ e−0.01x + Ke−0.01x = (1+ 0.01Kx ) ⋅ e−0.01x . The payment, given that a payment is made, with a deductible of d, is X* = ( X − d ) ( X > d ) . The survival function of that random variable is s X* ( x ) = Pr ( X* > x ) = Pr ( X − d > x X > d ) = Pr ( X > d + x X > d ) = = (1+ 0.01K ( x + d )) ⋅ e −0.01( x+d ) = (1+ 0.01Kd ) ⋅ e The expected value of ( X − 100 ) ( X > 100 ) is −0.01d +∞ 1+ 0.01K ( x + d ) −0.01x ⋅e . 1+ 0.01Kd +∞ 1+ K + 0.01Kx −0.01x 1 −0.01x ∫0 1+ K ⋅ e = 1+ K ⋅ ∫0 (1+ K + 0.01Kx ) ⋅ e = = u = 1+ K + 0.01Kx v = −100e−0.01x = du = 0.01Kdx dv = e−0.01x dx Integration by parts ⎛ 1+ K + 0.01Kx ⎞ = ⎜− ⋅100e−0.01x ⎟ ⎝ ⎠ 1+ K x→+∞ + x=0 +∞ K ⋅ 1+ K 100K dx = 100 + . 1+ K 0 ∫e −0.01x Mean of exponential with hazard rate of 0.01 1 Since this is equal to 125, we must have K = . Therefore, the expected value of 3 ( X − 200 ) ( X > 200 ) is +∞ −0.01x ∫ (1+ 0.002x ) ⋅ e .dx = 0 u = 1+ 0.002x v = −100e−0.01x = du = 0.002dx dv = e−0.01x dx Integration by parts = ( − (1+ 0.002x ) ⋅100e −0.01x ) x→+∞ x=0 +∞ + 0.2 ⋅ ∫e −0.01x dx = 100 + 0.2 ⋅100 = 120. 0 Mean of exponential with hazard rate of 0.01 Answer A. =